Regla de L’Hôpital para 0/0

Analizamos la regla de L’Hôpital para la indeterminación $0/0$.

RESUMEN TEÓRICO
  • El siguiente teorema proporciona un método para resolver en muchos casos, indeterminaciones del tipo $0/0.$
    Teorema  (Regla de L’Hôpital para $0/0$).  Sean $f$ y $g$ funciones derivables en el intervalo abierto $(a,b)$ tales que:
    $1)$ $\quad\displaystyle\lim_{x\to a^+}f(x)=\displaystyle\lim_{x\to a^+}g(x)=0$
    $2)\quad$ $f$ y $g$ son derivables en $(a,b)$ y $g'(x)\neq 0$ para todo $x\in (a,b).$
    $3)\quad$ Existe $\displaystyle\lim_{x\to a^+}\dfrac{f'(x)}{g'(x)}.$
    Entonces, $$\displaystyle\lim_{x\to a^+}\dfrac{f(x)}{g(x)}=\displaystyle\lim_{x\to a^+}\dfrac{f'(x)}{g'(x)}.$$
  • Observación.  Existe un teorema análogo cuando $x\to a^-,$ y combinando ambos, se obtiene otro, también análogo, cuando $x\to a.$
  • Ejemplo 1.  Hallemos $\;\displaystyle\lim_{x\to 0}\dfrac{\operatorname{sen}x}{2x}.$ Claramente, si $x\to 0,$ entonces $\operatorname{sen}x\to 0$ y $2x\to 0.$ El cociente de las derivadas es $\dfrac{\cos x}{2}$ cuyo límite es $\dfrac{1}{2}.$ Según la Regla de L’Hôpital: $$\lim_{x\to 0}\dfrac{\operatorname{sen}x}{2x}=\lim_{x\to 0}\dfrac{\cos x}{2}=\frac{1}{2}.$$ Convendremos en escribir abreviadamente los cálculos de la siguiente manera: $$\lim_{x\to 0}\dfrac{\operatorname{sen}x}{2x}=\left\{\frac{0}{0}\right\}=\lim_{x\to 0}\dfrac{\cos x}{2}=\frac{1}{2}.$$
  • Ejemplo 2.  Hallemos $\;L=\displaystyle\lim_{x\to 2}\dfrac{x^3-3x^2+4}{x^2-4x+4}.$ Tenemos
    $$L=\left\{\frac{0}{0}\right\}=\lim_{x\to 2}\dfrac{3x^2-6x}{2x-4}=\left\{\frac{0}{0}\right\}=\lim_{x\to 2}\dfrac{6x-6}{2}=\frac{6}{2}=3.$$
    Enunciado
  1. Calcular $L=\displaystyle\lim_{x\to 0}\dfrac{x+\operatorname{sen}2x}{x-\operatorname{sen}2x}.$
  2. Calcular $L=\displaystyle\lim_{x\to 0}\dfrac{e^x-1}{x^2}.$
  3. Calcular $L=\displaystyle\lim_{x\to 0}\dfrac{e^x+e^{-x}-x^2-2}{x^2-\operatorname{sen}^2x}.$
  4. Calcular $\displaystyle \lim_{x\to 0}\left(\frac{1}{\sin ^2x}-\frac{1}{x^2}\right)$.
  5. Demostrar la regla de L’Hôpital para $0/0$:
    Sean $f$ y $g$ funciones derivables en el intervalo abierto $(a,b)$ tales que:
    $1)$ $\quad\displaystyle\lim_{x\to a^+}f(x)=\displaystyle\lim_{x\to a^+}g(x)=0.$
    $2)\quad$ $f$ y $g$ son derivables en $(a,b)$ y $g'(x)\neq 0$ para todo $x\in (a,b).$
    $3)\quad$ Existe $\displaystyle\lim_{x\to a^+}\dfrac{f'(x)}{g'(x)}.$
    Entonces, $\displaystyle\lim_{x\to a^+}\dfrac{f(x)}{g(x)}=\displaystyle\lim_{x\to a^+}\dfrac{f'(x)}{g'(x)}.$
    Solución
  1. $ \quad L=\left\{\dfrac{0}{0}\right\}=\displaystyle\lim_{x\to 0}\dfrac{1+2\cos 2x}{1-2\cos2x}=\dfrac{1+2\cdot 1}{1-2\cdot 1}=-3.
    $
  2. $\quad L=\left\{\dfrac{0}{0}\right\}=\displaystyle\lim_{x\to 0}\dfrac{e^x}{2x}=\dfrac{1}{0}=\infty.
    $
    Nota. Podemos matizar: el límite es $+\infty$ si $x\to 0^+,$ y $-\infty$ si $x\to 0^-.$
  3. Tenemos: $$\begin{aligned}& L=\left\{\frac{0}{0}\right\}=\displaystyle\lim_{x\to 0}\dfrac{e^x-e^{-x}-2x}{2x-2\operatorname{sen}x\cos x}=\displaystyle\lim_{x\to 0}\dfrac{e^x-e^{-x}-2x}{2x-\operatorname{sen}2x}=\left\{\frac{0}{0}\right\}\\
    &=\displaystyle\lim_{x\to 0}\dfrac{e^x+e^{-x}-2}{2-2\cos2x}=\left\{\frac{0}{0}\right\}=\displaystyle\lim_{x\to 0}\dfrac{e^x-e^{-x}}{4\operatorname{sen}2x}=\left\{\frac{0}{0}\right\}=\displaystyle\lim_{x\to 0}\dfrac{e^x+e^{-x}}{8\cos2x}\\
    &=\dfrac{2}{8}=\dfrac{1}{4}.
    \end{aligned}$$
  4. Tenemos $$\lim_{x\to 0}\left(\frac{1}{\sin ^2x}-\frac{1}{x^2}\right)=\lim_{x\to 0}\frac{x^2-\sin ^2 x}{x^2\sin ^2 x}=\left\{\frac{0}{0}\right\}\underbrace{=}_{\sin x\sim x}\lim_{x\to 0}\frac{x^2-\sin ^2 x}{x^4}$$ $$\underbrace{=}_{\text{L’Hop.}}\lim_{x\to 0}\frac{2x-2\sin x\cos x}{4x^3}=\lim_{x\to 0}\frac{2x-\sin 2x}{4x^3}=\left\{\frac{0}{0}\right\}\underbrace{=}_{\text{L’Hop.}}\lim_{x\to 0}\frac{2-2\cos 2x }{12x^2}$$ $$=\left\{\frac{0}{0}\right\}\underbrace{=}_{\text{L’Hop.}}\lim_{x\to 0}\frac{4\sin 2x }{24x}=\left\{\frac{0}{0}\right\}\underbrace{=}_{\text{L’Hop.}}\lim_{x\to 0}\frac{8\cos 2x }{24}=\frac{8}{24}=\frac{1}{3}.$$
  5. Consideremos las funciones: $$F(x)=\left \{ \begin{matrix} f(x)& \mbox{ si }& x\in (a,b)\\0 & \mbox{ si }&x=a\end{matrix}\right.\;,\quad G(x)=\left \{ \begin{matrix} g(x)& \mbox{ si }& x\in (a,b)\\0 & \mbox{ si }&x=a.\end{matrix}\right.$$ Por la hipótesis $1),$ $F$ y $G$ son continuas en $a$ y por la hipótesis $3),$ son derivables en $(a,b)$ con $G'(x)\neq 0$ para todo $x\in (a,b).$ Esto implica que se verifican las hipótesis del teorema del valor medio de Lagrange para las funciones $F$ y $G$ en todo intervalo cerrado $[a,x]$ con $a<x<b.$ Existe por tanto un $c\in (a,x)$ tal que: $$\frac{F'(c)}{G'(c)}=\frac{F(x)-F(a)}{G(x)-G(a)}.$$ Según se han definido $F$ y $G,$ la anterior igualdad equivale a: $$\frac{f'(c)}{g'(c)}=\frac{f(x)}{g(x)}.$$ Si $x\to a^+,$ entonces $c\to a^+,$ pues $a<c<x.$ Dado que por hipótesis existe el límite de $3),$ se verifica: $$\displaystyle\lim_{x\to a^+}\dfrac{f(x)}{g(x)}=\displaystyle\lim_{x\to a^+}\dfrac{f'(x)}{g'(x)}.$$
Esta entrada ha sido publicada en Análisis real y complejo y etiquetada como , . Guarda el enlace permanente.